Find an ellipse centered through the origin that runs through 3 points

Click For Summary
To find the ellipse centered at the origin that passes through the points (1,2), (2,2), and (3,1), the equation can be expressed in the form ax^2 + bxy + cy^2 = 1. The terms ax^2 and cy^2 correspond to the standard ellipse equation, while the bxy term allows for the ellipse to be rotated, accommodating more general cases. By substituting the coordinates of the three points into the equation, a system of equations can be established to solve for the unknown coefficients a, b, and c. This approach highlights the flexibility of the general ellipse equation compared to the standard form. Ultimately, the solution will yield the specific parameters defining the desired ellipse.
mahrap
Messages
37
Reaction score
0
Find the ellipse centered at the origin that runs through
the points (1,2), (2,2), and (3, I). Write your equation
in the form $$ ax^2 + bxy + cy^2 = 1 $$

I understand the $$ ax^2 $$ and $$ cy^2 $$ in the equation because the equation of an ellipse centered at origin is $$ (x/a)^2 + (y/b)^2 = 1 $$ so we let $$ a = (1/a)^2 $$ and $$ b = (1/b)^2 $$. but where did the $$ bxy $$ come from?
 
Physics news on Phys.org
mahrap said:
Find the ellipse centered at the origin that runs through
the points (1,2), (2,2), and (3, I). Write your equation
in the form $$ ax^2 + bxy + cy^2 = 1 $$

I understand the $$ ax^2 $$ and $$ cy^2 $$ in the equation because the equation of an ellipse centered at origin is $$ (x/a)^2 + (y/b)^2 = 1 $$ so we let $$ a = (1/a)^2 $$ and $$ b = (1/b)^2 $$. but where did the $$ bxy $$ come from?

##(x/a)^2 + (y/b)^2 = 1## is only the equation of an ellipse centered at the origin whose axes are parallel to x and y axes. ##ax^2 + bxy + cy^2 = 1## is more general. It may be at an angle. Just put the given values for x and y in and get three equations to solve for the three unknowns a, b and c.
 
Question: A clock's minute hand has length 4 and its hour hand has length 3. What is the distance between the tips at the moment when it is increasing most rapidly?(Putnam Exam Question) Answer: Making assumption that both the hands moves at constant angular velocities, the answer is ## \sqrt{7} .## But don't you think this assumption is somewhat doubtful and wrong?

Similar threads

  • · Replies 4 ·
Replies
4
Views
3K
  • · Replies 2 ·
Replies
2
Views
3K
  • · Replies 5 ·
Replies
5
Views
2K
  • · Replies 1 ·
Replies
1
Views
2K
  • · Replies 9 ·
Replies
9
Views
2K
  • · Replies 4 ·
Replies
4
Views
2K
  • · Replies 8 ·
Replies
8
Views
2K
  • · Replies 9 ·
Replies
9
Views
4K
  • · Replies 8 ·
Replies
8
Views
3K
Replies
4
Views
3K